LSAT and Law School Admissions Forum

Get expert LSAT preparation and law school admissions advice from PowerScore Test Preparation.

 amydg
  • Posts: 6
  • Joined: May 21, 2016
|
#25729
I was stuck between B and C and I thought B provided a more specific flaw in the reasoning since it talked about unreliable sources, so why is C the correct answer? Thanks :-?
 Nikki Siclunov
PowerScore Staff
  • PowerScore Staff
  • Posts: 1362
  • Joined: Aug 02, 2011
|
#25787
Hi Amy,

Thanks for your question! Here’s what the argument boils down to:

Union leaders oppose multinational control, because it has apparently led to a decrease in global wages. But, the author counters, these leaders have a vested interest in keeping wages high. So, the author urges legislators to reject the union leaders’ argument.

This is a classic Error in the Use of Evidence. Even if the author weakened the union leader’s argument, by suggesting a potentially biased source of information, that doesn’t mean that their conclusion should be flat-out rejected. There may still be a good reason for legislators to oppose multinational control despite a circumstance that may affect the union leaders' judgment. In other words, the author confuses undermining an argument in support of a given position with proving that position to be false. Answer choices (B) and (C) are indeed the closest to that particular prephrase.

The problem with answer choice (B) is that it alludes to an assumption, and assumptions are difficult to prove: they need to pass an exceptionally high bar (otherwise known as the Assumption Negation Technique). Does the argument really assume that anyone whose political motivations are clearly discernible is an unreliable source of information to legislators? Not necessarily. The argument presumes that the union leaders are an unreliable source of information to legislators, in this particular case. Try negating the alleged assumption here: what if some people whose political motivations are clearly discernible may still represent a reliable source of information to legislators? Does that weaken the politician's position? Not really. He can easily respond by saying, well, yes - some people may be reliable, but the union leaders aren't. Screw those guys!

Simply put, answer choice (B) goes too far, alleging an assumption that the argument ultimately doesn’t rely upon. For me to accept answer choice (B) as correct, I would need to reword it as follows: “… presumes, without justification, that clearly discernible political motivations could make one an unreliable source of information to legislators.” Do you see how minor variations in language can render an assumption answer correct (or incorrect)?

Unlike (B), we can easily prove answer choice (C). The author treats a circumstance that may potentially bias the opposing argument (the union leaders have a vested interest in keeping wages high) as sufficient to discredit that argument. “Discredit” is not too strong of a word, considering the definitive nature of the politician’s conclusion (“Thus, legislators should reject this argument”).

Never forget that Flaw in the Reasoning questions ultimately belong to the Prove family of questions. If you can’t prove that the author made the unwarranted assumption suggested in the answer choice (typically introduced with “presumes, without providing justification,” or “takes for granted that…”), then the answer choice is incorrect.

Hope this helps!

Nikki
 mokkyukkyu
  • Posts: 97
  • Joined: Aug 17, 2016
|
#28761
Let me double check whether I understand this question correctly...
So (D) is wrong because it only says "whose motivations are clearly discernible" but this is not how the author denies their argument and tells to reject it. Rather, the author says because they have "a vested interest", they should reject it. So not ANY motivations, but THE SPECIFIC motivation makes their argument doubtful---thus (B) is wrong. (C) is right because it mentions THE SPECIFIC motivation---the circumstance that may affect why the leaders make the argument. Thus, C is the correct answer. Am I correct?
 ellyb
  • Posts: 16
  • Joined: Aug 29, 2016
|
#28840
can someone please explain why D is wrong? i've looked this up all over the internet and people aren't reconciling the problem i have with this answer choice.

If the politician assumes that this is the only argument they have for their view, then you're neglecting the possibility that they have other arguments, other arguments that may LEGITIMATELY explain their point of view, thus making it incorrect to discredit the argument based on just one of the views when that view wasn't the main catalyst for the conclusion anyway.
 Nikki Siclunov
PowerScore Staff
  • PowerScore Staff
  • Posts: 1362
  • Joined: Aug 02, 2011
|
#28847
Hey guys,

I think you are both asking the same question, i.e. what makes answer choice (D) incorrect. I hope you both understood, from my explanation above, why answer choice (C) is correct - it correctly captures the Error in the Use of Evidence, i.e. the erroneous belief that weakening an argument in support of a given position actually disproves this argument.

Answer choice (D) outlines a possible assumption ("presumes..."), namely, that the argument cited is the union leaders' only argument for their view. Does the author need to assume that the union leaders haven't made any other arguments defending their view? Of course not. Even if they have made other arguments, that wouldn't weaken the politician's conclusion. After all, the union leaders are biased, says the politician, so whatever arguments they make can theoretically be dismissed for the same reason as the argument cited in the stimulus. Since the logical opposite of the alleged assumption in answer choice (D) does not weaken the politician's argument, answer choice (D) does not contain an assumption upon which his argument depends.

ellyb - your approach to answer choice (D) is incorrect. This is not a Weaken question - we can't take it for granted that the politician actually made this assumption. Rather, for answer choice (D) to be correct, you need to prove that this assumption was, in fact, made. It was not.


Hope this helps! :)

Thanks,
 sgd2114
  • Posts: 23
  • Joined: Jul 14, 2017
|
#39394
Hi -

I thought of the flaw as a Source Attack - using the union leaders' potential motivation / affiliation as grounds to dismiss their argument. I see now the Error in Use of Evidence - i.e., some evidence against used to prove false. However, is (A) wrong because it says "to case doubt on ALL of the viewpoints expressed by those people?" Rather, the stimulus is just using the fact that these people are union members as sufficient to cast doubt on this claim (not all viewpoints)? Or, is it justified to cast doubt on the union members based on their affiliation?

Thank you!
 James Finch
PowerScore Staff
  • PowerScore Staff
  • Posts: 943
  • Joined: Sep 06, 2017
|
#39760
Hi SGD,

(A) starts off by giving us something new, something not present in the stimulus: it talks about union members, as opposed to the union leaders discussed in the stimulus. It also says that that all of their viewpoints should be called into question--is this supported by the stimulus? What if a union member argued that kittens were cute, would this politician argue that we shouldn't believe them?

Moreover, it's not the fact that the union leaders are members of a union that is being used to discredit their arguments, it's that they have an alleged vested interest in high wages--what we don't have is why that vested interest would mean that legislators should oppose the union leaders' arguments.

Hope this clears things up!
 lanereuden
  • Posts: 147
  • Joined: May 30, 2019
|
#67489
Nikki Siclunov wrote:Hi Amy,

Thanks for your question! Here’s what the argument boils down to:

Union leaders oppose multinational control, because it has apparently led to a decrease in global wages. But, the author counters, these leaders have a vested interest in keeping wages high. So, the author urges legislators to reject the union leaders’ argument.

This is a classic Error in the Use of Evidence. Even if the author weakened the union leader’s argument, by suggesting a potentially biased source of information, that doesn’t mean that their conclusion should be flat-out rejected. There may still be a good reason for legislators to oppose multinational control despite a circumstance that may affect the union leaders' judgment. In other words, the author confuses undermining an argument in support of a given position with proving that position to be false. Answer choices (B) and (C) are indeed the closest to that particular prephrase.

The problem with answer choice (B) is that it alludes to an assumption, and assumptions are difficult to prove: they need to pass an exceptionally high bar (otherwise known as the Assumption Negation Technique). Does the argument really assume that anyone whose political motivations are clearly discernible is an unreliable source of information to legislators? Not necessarily. The argument presumes that the union leaders are an unreliable source of information to legislators, in this particular case. Try negating the alleged assumption here: what if some people whose political motivations are clearly discernible may still represent a reliable source of information to legislators? Does that weaken the politician's position? Not really. He can easily respond by saying, well, yes - some people may be reliable, but the union leaders aren't. Screw those guys!

Simply put, answer choice (B) goes too far, alleging an assumption that the argument ultimately doesn’t rely upon. For me to accept answer choice (B) as correct, I would need to reword it as follows: “… presumes, without justification, that clearly discernible political motivations could make one an unreliable source of information to legislators.” Do you see how minor variations in language can render an assumption answer correct (or incorrect)?

Unlike (B), we can easily prove answer choice (C). The author treats a circumstance that may potentially bias the opposing argument (the union leaders have a vested interest in keeping wages high) as sufficient to discredit that argument. “Discredit” is not too strong of a word, considering the definitive nature of the politician’s conclusion (“Thus, legislators should reject this argument”).

Never forget that Flaw in the Reasoning questions ultimately belong to the Prove family of questions. If you can’t prove that the author made the unwarranted assumption suggested in the answer choice (typically introduced with “presumes, without providing justification,” or “takes for granted that…”), then the answer choice is incorrect.

Hope this helps!

Nikki
I thought B was wrong not because it should have read:
could make one an unreliable source of information to legislators
But rather because it should have read : anyone whose p.m. are Cleary discernible is sufficient to discredit argument
Thoughts?
 2020//Vision
  • Posts: 6
  • Joined: Sep 25, 2020
|
#79344
For this flaw question, why is this not a source/ad hominem flaw? The conclusion states legislators should reject the union leaders' argument -- on the basis of their motives. How is that not an ad hominem/source argument flaw? (Course Lesson 7?)
 Jeremy Press
PowerScore Staff
  • PowerScore Staff
  • Posts: 1000
  • Joined: Jun 12, 2017
|
#79963
Hi 2020,

Nikki is absolutely right above that there is an "Evidence Error" problem here--that just because there are some reasons to discredit the union leaders, that doesn't mean the leaders' argument is false. But there's definitely a subtle form of "attack on the source" that's going on here as well! It's just that, in this case, there is some reason to be a little skeptical of what the union leaders are saying. Their motive to favor the argument they're giving means we should be careful with what they say. The description of the flaw that is given in answer choice C fits very well with what we know of this kind of subtle attack on motives--circumstances where there is a motive that is being attacked that should make us skeptical without necessarily rendering someone's argument false. As answer choice C says, the union leaders' motive to ensure that wages remain high does potentially affect their argument. Motives often have an effect on the arguments we make. If you ask me, an employee, whether policies benefiting employees should be adopted, I'm of course going to have a motive to support those policies. And that motive is likely going to make my argument more strongly and sharply in favor of those policies (i.e., the motive is going to have an effect on my argument). So you should be a little skeptical of what I'm saying, because bias might be infecting my ideas. But does that mean my entire argument is false and should be rejected? Of course not! I might have some really good policy ideas that go along with my motives. Same with our union leaders here. They have some motives that are very likely affecting the argument they're offering. So we should be a little skeptical of them. But just pointing out those motives doesn't by itself mean the union leaders' argument should be rejected. In other words, those motives aren't sufficient to discredit their argument. That's more of an Evidence Error than the classic "ad hominem," because in a classic ad hominem, the actions or characteristics of the source that's being attacked don't have anything to do with the argument or its ideas. Here, the motives of the union leaders do give us a little reason to be skeptical of their argument, but those motives should not be treated as determinative.

I hope this helps!

Get the most out of your LSAT Prep Plus subscription.

Analyze and track your performance with our Testing and Analytics Package.